Con(ZFC)

http://d.hatena.ne.jp/gin_math/20050606/p2
> 完全性定理によりZFC+\ne Con(ZFC)のモデルMが存在する。となると、Mの中には有限個のZFCの公理でそこから矛盾が証明できるようなものが存在することになる。
ここの展開が"恐らく"おかしいですね。もしも、示せたならばクレイ数学研究所に駆け込んで賞金を全部もらいませう。